Determine all triples $(m,n,p)$ of positive rational numbers.












0












$begingroup$


Determine all triples $(m,n,p)$ of positive rational numbers such that the numbers $m+frac{1}{np}, n+frac{1}{pm}, p+frac{1}{mn}$ are integers.



I have no idea how to go about. Please help.










share|cite|improve this question











$endgroup$












  • $begingroup$
    $m+frac{1}{n}p$ or $m+frac{1}{np}$?
    $endgroup$
    – Dietrich Burde
    Jan 12 at 16:42










  • $begingroup$
    The latter one.
    $endgroup$
    – Yellow
    Jan 12 at 16:45










  • $begingroup$
    @Yellow, could we speak in chat? chat.stackexchange.com/rooms/88182/room-for-haran-and-yellow
    $endgroup$
    – Haran
    Jan 12 at 17:19










  • $begingroup$
    @Yellow are you online?
    $endgroup$
    – Haran
    Jan 13 at 5:08
















0












$begingroup$


Determine all triples $(m,n,p)$ of positive rational numbers such that the numbers $m+frac{1}{np}, n+frac{1}{pm}, p+frac{1}{mn}$ are integers.



I have no idea how to go about. Please help.










share|cite|improve this question











$endgroup$












  • $begingroup$
    $m+frac{1}{n}p$ or $m+frac{1}{np}$?
    $endgroup$
    – Dietrich Burde
    Jan 12 at 16:42










  • $begingroup$
    The latter one.
    $endgroup$
    – Yellow
    Jan 12 at 16:45










  • $begingroup$
    @Yellow, could we speak in chat? chat.stackexchange.com/rooms/88182/room-for-haran-and-yellow
    $endgroup$
    – Haran
    Jan 12 at 17:19










  • $begingroup$
    @Yellow are you online?
    $endgroup$
    – Haran
    Jan 13 at 5:08














0












0








0


1



$begingroup$


Determine all triples $(m,n,p)$ of positive rational numbers such that the numbers $m+frac{1}{np}, n+frac{1}{pm}, p+frac{1}{mn}$ are integers.



I have no idea how to go about. Please help.










share|cite|improve this question











$endgroup$




Determine all triples $(m,n,p)$ of positive rational numbers such that the numbers $m+frac{1}{np}, n+frac{1}{pm}, p+frac{1}{mn}$ are integers.



I have no idea how to go about. Please help.







elementary-number-theory rational-numbers






share|cite|improve this question















share|cite|improve this question













share|cite|improve this question




share|cite|improve this question








edited Jan 13 at 5:04









Haran

1,076323




1,076323










asked Jan 12 at 16:39









YellowYellow

16011




16011












  • $begingroup$
    $m+frac{1}{n}p$ or $m+frac{1}{np}$?
    $endgroup$
    – Dietrich Burde
    Jan 12 at 16:42










  • $begingroup$
    The latter one.
    $endgroup$
    – Yellow
    Jan 12 at 16:45










  • $begingroup$
    @Yellow, could we speak in chat? chat.stackexchange.com/rooms/88182/room-for-haran-and-yellow
    $endgroup$
    – Haran
    Jan 12 at 17:19










  • $begingroup$
    @Yellow are you online?
    $endgroup$
    – Haran
    Jan 13 at 5:08


















  • $begingroup$
    $m+frac{1}{n}p$ or $m+frac{1}{np}$?
    $endgroup$
    – Dietrich Burde
    Jan 12 at 16:42










  • $begingroup$
    The latter one.
    $endgroup$
    – Yellow
    Jan 12 at 16:45










  • $begingroup$
    @Yellow, could we speak in chat? chat.stackexchange.com/rooms/88182/room-for-haran-and-yellow
    $endgroup$
    – Haran
    Jan 12 at 17:19










  • $begingroup$
    @Yellow are you online?
    $endgroup$
    – Haran
    Jan 13 at 5:08
















$begingroup$
$m+frac{1}{n}p$ or $m+frac{1}{np}$?
$endgroup$
– Dietrich Burde
Jan 12 at 16:42




$begingroup$
$m+frac{1}{n}p$ or $m+frac{1}{np}$?
$endgroup$
– Dietrich Burde
Jan 12 at 16:42












$begingroup$
The latter one.
$endgroup$
– Yellow
Jan 12 at 16:45




$begingroup$
The latter one.
$endgroup$
– Yellow
Jan 12 at 16:45












$begingroup$
@Yellow, could we speak in chat? chat.stackexchange.com/rooms/88182/room-for-haran-and-yellow
$endgroup$
– Haran
Jan 12 at 17:19




$begingroup$
@Yellow, could we speak in chat? chat.stackexchange.com/rooms/88182/room-for-haran-and-yellow
$endgroup$
– Haran
Jan 12 at 17:19












$begingroup$
@Yellow are you online?
$endgroup$
– Haran
Jan 13 at 5:08




$begingroup$
@Yellow are you online?
$endgroup$
– Haran
Jan 13 at 5:08










1 Answer
1






active

oldest

votes


















2












$begingroup$

First, by multiplying, we can see that $frac{(mnp+1)^3}{(mnp)^2}$ is an integer. Consider $mnp = frac{X}{Y}$, where $gcd(X,Y)=1$. Then, you can see that $X=Y=1$. This gives you $mnp=1$. Replace $m=frac{w}{x}$, $n=frac{y}{z}$, $p=frac{xz}{wy}$, we can see using the three statements with divisibility solving that the only solutions are $(1,1,1)$ , $(2,1,frac{1}{2})$ and $(4,frac{1}{2},frac{1}{2})$, and all its symmetric permutations.



The statements we get are $x mid 2w$, $z mid 2y$ and $wy mid 2xz$. You can take casework to solve these.






share|cite|improve this answer









$endgroup$













  • $begingroup$
    @Yellow can we speak in private chat?
    $endgroup$
    – Haran
    Jan 12 at 16:58






  • 2




    $begingroup$
    To add, this problem is from 2006 Balkan Mathematical Olympiad, proposed by Valentin Vornicu, and the solution outlined above, as far as I remember, goes back to Gabriel Dospinescu (aka harazi in AoPS forums).
    $endgroup$
    – Aaron
    Jan 12 at 17:45










  • $begingroup$
    Haran, in nowhere I wrote, I did say it is not yours. I said this outline is quite old, goes back to Harazi.
    $endgroup$
    – Aaron
    Jan 12 at 17:54










  • $begingroup$
    See AoPS (www.mathlinks.ro) forums, he's a member there, and had many questions in Romanian and USAMO olympiads. In fact, if you are into olympiad mathematics, I highly recommend that you get an account from there, it has a rich collection of many olympiad problems from all across the world.
    $endgroup$
    – Aaron
    Jan 12 at 18:04












  • $begingroup$
    I already have an account Aaron. Thanks for the recommendation though!
    $endgroup$
    – Haran
    Jan 12 at 18:05











Your Answer





StackExchange.ifUsing("editor", function () {
return StackExchange.using("mathjaxEditing", function () {
StackExchange.MarkdownEditor.creationCallbacks.add(function (editor, postfix) {
StackExchange.mathjaxEditing.prepareWmdForMathJax(editor, postfix, [["$", "$"], ["\\(","\\)"]]);
});
});
}, "mathjax-editing");

StackExchange.ready(function() {
var channelOptions = {
tags: "".split(" "),
id: "69"
};
initTagRenderer("".split(" "), "".split(" "), channelOptions);

StackExchange.using("externalEditor", function() {
// Have to fire editor after snippets, if snippets enabled
if (StackExchange.settings.snippets.snippetsEnabled) {
StackExchange.using("snippets", function() {
createEditor();
});
}
else {
createEditor();
}
});

function createEditor() {
StackExchange.prepareEditor({
heartbeatType: 'answer',
autoActivateHeartbeat: false,
convertImagesToLinks: true,
noModals: true,
showLowRepImageUploadWarning: true,
reputationToPostImages: 10,
bindNavPrevention: true,
postfix: "",
imageUploader: {
brandingHtml: "Powered by u003ca class="icon-imgur-white" href="https://imgur.com/"u003eu003c/au003e",
contentPolicyHtml: "User contributions licensed under u003ca href="https://creativecommons.org/licenses/by-sa/3.0/"u003ecc by-sa 3.0 with attribution requiredu003c/au003e u003ca href="https://stackoverflow.com/legal/content-policy"u003e(content policy)u003c/au003e",
allowUrls: true
},
noCode: true, onDemand: true,
discardSelector: ".discard-answer"
,immediatelyShowMarkdownHelp:true
});


}
});














draft saved

draft discarded


















StackExchange.ready(
function () {
StackExchange.openid.initPostLogin('.new-post-login', 'https%3a%2f%2fmath.stackexchange.com%2fquestions%2f3071097%2fdetermine-all-triples-m-n-p-of-positive-rational-numbers%23new-answer', 'question_page');
}
);

Post as a guest















Required, but never shown

























1 Answer
1






active

oldest

votes








1 Answer
1






active

oldest

votes









active

oldest

votes






active

oldest

votes









2












$begingroup$

First, by multiplying, we can see that $frac{(mnp+1)^3}{(mnp)^2}$ is an integer. Consider $mnp = frac{X}{Y}$, where $gcd(X,Y)=1$. Then, you can see that $X=Y=1$. This gives you $mnp=1$. Replace $m=frac{w}{x}$, $n=frac{y}{z}$, $p=frac{xz}{wy}$, we can see using the three statements with divisibility solving that the only solutions are $(1,1,1)$ , $(2,1,frac{1}{2})$ and $(4,frac{1}{2},frac{1}{2})$, and all its symmetric permutations.



The statements we get are $x mid 2w$, $z mid 2y$ and $wy mid 2xz$. You can take casework to solve these.






share|cite|improve this answer









$endgroup$













  • $begingroup$
    @Yellow can we speak in private chat?
    $endgroup$
    – Haran
    Jan 12 at 16:58






  • 2




    $begingroup$
    To add, this problem is from 2006 Balkan Mathematical Olympiad, proposed by Valentin Vornicu, and the solution outlined above, as far as I remember, goes back to Gabriel Dospinescu (aka harazi in AoPS forums).
    $endgroup$
    – Aaron
    Jan 12 at 17:45










  • $begingroup$
    Haran, in nowhere I wrote, I did say it is not yours. I said this outline is quite old, goes back to Harazi.
    $endgroup$
    – Aaron
    Jan 12 at 17:54










  • $begingroup$
    See AoPS (www.mathlinks.ro) forums, he's a member there, and had many questions in Romanian and USAMO olympiads. In fact, if you are into olympiad mathematics, I highly recommend that you get an account from there, it has a rich collection of many olympiad problems from all across the world.
    $endgroup$
    – Aaron
    Jan 12 at 18:04












  • $begingroup$
    I already have an account Aaron. Thanks for the recommendation though!
    $endgroup$
    – Haran
    Jan 12 at 18:05
















2












$begingroup$

First, by multiplying, we can see that $frac{(mnp+1)^3}{(mnp)^2}$ is an integer. Consider $mnp = frac{X}{Y}$, where $gcd(X,Y)=1$. Then, you can see that $X=Y=1$. This gives you $mnp=1$. Replace $m=frac{w}{x}$, $n=frac{y}{z}$, $p=frac{xz}{wy}$, we can see using the three statements with divisibility solving that the only solutions are $(1,1,1)$ , $(2,1,frac{1}{2})$ and $(4,frac{1}{2},frac{1}{2})$, and all its symmetric permutations.



The statements we get are $x mid 2w$, $z mid 2y$ and $wy mid 2xz$. You can take casework to solve these.






share|cite|improve this answer









$endgroup$













  • $begingroup$
    @Yellow can we speak in private chat?
    $endgroup$
    – Haran
    Jan 12 at 16:58






  • 2




    $begingroup$
    To add, this problem is from 2006 Balkan Mathematical Olympiad, proposed by Valentin Vornicu, and the solution outlined above, as far as I remember, goes back to Gabriel Dospinescu (aka harazi in AoPS forums).
    $endgroup$
    – Aaron
    Jan 12 at 17:45










  • $begingroup$
    Haran, in nowhere I wrote, I did say it is not yours. I said this outline is quite old, goes back to Harazi.
    $endgroup$
    – Aaron
    Jan 12 at 17:54










  • $begingroup$
    See AoPS (www.mathlinks.ro) forums, he's a member there, and had many questions in Romanian and USAMO olympiads. In fact, if you are into olympiad mathematics, I highly recommend that you get an account from there, it has a rich collection of many olympiad problems from all across the world.
    $endgroup$
    – Aaron
    Jan 12 at 18:04












  • $begingroup$
    I already have an account Aaron. Thanks for the recommendation though!
    $endgroup$
    – Haran
    Jan 12 at 18:05














2












2








2





$begingroup$

First, by multiplying, we can see that $frac{(mnp+1)^3}{(mnp)^2}$ is an integer. Consider $mnp = frac{X}{Y}$, where $gcd(X,Y)=1$. Then, you can see that $X=Y=1$. This gives you $mnp=1$. Replace $m=frac{w}{x}$, $n=frac{y}{z}$, $p=frac{xz}{wy}$, we can see using the three statements with divisibility solving that the only solutions are $(1,1,1)$ , $(2,1,frac{1}{2})$ and $(4,frac{1}{2},frac{1}{2})$, and all its symmetric permutations.



The statements we get are $x mid 2w$, $z mid 2y$ and $wy mid 2xz$. You can take casework to solve these.






share|cite|improve this answer









$endgroup$



First, by multiplying, we can see that $frac{(mnp+1)^3}{(mnp)^2}$ is an integer. Consider $mnp = frac{X}{Y}$, where $gcd(X,Y)=1$. Then, you can see that $X=Y=1$. This gives you $mnp=1$. Replace $m=frac{w}{x}$, $n=frac{y}{z}$, $p=frac{xz}{wy}$, we can see using the three statements with divisibility solving that the only solutions are $(1,1,1)$ , $(2,1,frac{1}{2})$ and $(4,frac{1}{2},frac{1}{2})$, and all its symmetric permutations.



The statements we get are $x mid 2w$, $z mid 2y$ and $wy mid 2xz$. You can take casework to solve these.







share|cite|improve this answer












share|cite|improve this answer



share|cite|improve this answer










answered Jan 12 at 16:56









HaranHaran

1,076323




1,076323












  • $begingroup$
    @Yellow can we speak in private chat?
    $endgroup$
    – Haran
    Jan 12 at 16:58






  • 2




    $begingroup$
    To add, this problem is from 2006 Balkan Mathematical Olympiad, proposed by Valentin Vornicu, and the solution outlined above, as far as I remember, goes back to Gabriel Dospinescu (aka harazi in AoPS forums).
    $endgroup$
    – Aaron
    Jan 12 at 17:45










  • $begingroup$
    Haran, in nowhere I wrote, I did say it is not yours. I said this outline is quite old, goes back to Harazi.
    $endgroup$
    – Aaron
    Jan 12 at 17:54










  • $begingroup$
    See AoPS (www.mathlinks.ro) forums, he's a member there, and had many questions in Romanian and USAMO olympiads. In fact, if you are into olympiad mathematics, I highly recommend that you get an account from there, it has a rich collection of many olympiad problems from all across the world.
    $endgroup$
    – Aaron
    Jan 12 at 18:04












  • $begingroup$
    I already have an account Aaron. Thanks for the recommendation though!
    $endgroup$
    – Haran
    Jan 12 at 18:05


















  • $begingroup$
    @Yellow can we speak in private chat?
    $endgroup$
    – Haran
    Jan 12 at 16:58






  • 2




    $begingroup$
    To add, this problem is from 2006 Balkan Mathematical Olympiad, proposed by Valentin Vornicu, and the solution outlined above, as far as I remember, goes back to Gabriel Dospinescu (aka harazi in AoPS forums).
    $endgroup$
    – Aaron
    Jan 12 at 17:45










  • $begingroup$
    Haran, in nowhere I wrote, I did say it is not yours. I said this outline is quite old, goes back to Harazi.
    $endgroup$
    – Aaron
    Jan 12 at 17:54










  • $begingroup$
    See AoPS (www.mathlinks.ro) forums, he's a member there, and had many questions in Romanian and USAMO olympiads. In fact, if you are into olympiad mathematics, I highly recommend that you get an account from there, it has a rich collection of many olympiad problems from all across the world.
    $endgroup$
    – Aaron
    Jan 12 at 18:04












  • $begingroup$
    I already have an account Aaron. Thanks for the recommendation though!
    $endgroup$
    – Haran
    Jan 12 at 18:05
















$begingroup$
@Yellow can we speak in private chat?
$endgroup$
– Haran
Jan 12 at 16:58




$begingroup$
@Yellow can we speak in private chat?
$endgroup$
– Haran
Jan 12 at 16:58




2




2




$begingroup$
To add, this problem is from 2006 Balkan Mathematical Olympiad, proposed by Valentin Vornicu, and the solution outlined above, as far as I remember, goes back to Gabriel Dospinescu (aka harazi in AoPS forums).
$endgroup$
– Aaron
Jan 12 at 17:45




$begingroup$
To add, this problem is from 2006 Balkan Mathematical Olympiad, proposed by Valentin Vornicu, and the solution outlined above, as far as I remember, goes back to Gabriel Dospinescu (aka harazi in AoPS forums).
$endgroup$
– Aaron
Jan 12 at 17:45












$begingroup$
Haran, in nowhere I wrote, I did say it is not yours. I said this outline is quite old, goes back to Harazi.
$endgroup$
– Aaron
Jan 12 at 17:54




$begingroup$
Haran, in nowhere I wrote, I did say it is not yours. I said this outline is quite old, goes back to Harazi.
$endgroup$
– Aaron
Jan 12 at 17:54












$begingroup$
See AoPS (www.mathlinks.ro) forums, he's a member there, and had many questions in Romanian and USAMO olympiads. In fact, if you are into olympiad mathematics, I highly recommend that you get an account from there, it has a rich collection of many olympiad problems from all across the world.
$endgroup$
– Aaron
Jan 12 at 18:04






$begingroup$
See AoPS (www.mathlinks.ro) forums, he's a member there, and had many questions in Romanian and USAMO olympiads. In fact, if you are into olympiad mathematics, I highly recommend that you get an account from there, it has a rich collection of many olympiad problems from all across the world.
$endgroup$
– Aaron
Jan 12 at 18:04














$begingroup$
I already have an account Aaron. Thanks for the recommendation though!
$endgroup$
– Haran
Jan 12 at 18:05




$begingroup$
I already have an account Aaron. Thanks for the recommendation though!
$endgroup$
– Haran
Jan 12 at 18:05


















draft saved

draft discarded




















































Thanks for contributing an answer to Mathematics Stack Exchange!


  • Please be sure to answer the question. Provide details and share your research!

But avoid



  • Asking for help, clarification, or responding to other answers.

  • Making statements based on opinion; back them up with references or personal experience.


Use MathJax to format equations. MathJax reference.


To learn more, see our tips on writing great answers.




draft saved


draft discarded














StackExchange.ready(
function () {
StackExchange.openid.initPostLogin('.new-post-login', 'https%3a%2f%2fmath.stackexchange.com%2fquestions%2f3071097%2fdetermine-all-triples-m-n-p-of-positive-rational-numbers%23new-answer', 'question_page');
}
);

Post as a guest















Required, but never shown





















































Required, but never shown














Required, but never shown












Required, but never shown







Required, but never shown

































Required, but never shown














Required, but never shown












Required, but never shown







Required, but never shown







Popular posts from this blog

Can a sorcerer learn a 5th-level spell early by creating spell slots using the Font of Magic feature?

Does disintegrating a polymorphed enemy still kill it after the 2018 errata?

A Topological Invariant for $pi_3(U(n))$